Algebra II, Take Home Exam to Be Submitted on Canvas by 10:00 P.M

Algebra II, Take Home Exam to Be Submitted on Canvas by 10:00 P.M

Algebra II, Take Home Exam To be submitted on Canvas by 10:00 P.M. on Wednesday, May 6. You may use material from Jacobson and from the class notes in your solutions to these problems. If you use a result from these sources you should cite it (e.g. \The proposition on slide 6 of Lecture 5"). You may not consult with other people or use other references. 1) Let A be a commutative ring in which every element x satisfies xn = x for some n > 1 (depending on x). Show that every prime ideal in A is maximal. Solution. Suppose that P is a prime ideal of R. For x 2 R, let x be the class of x in R=P . Suppose x 6= 0. xn = x for some n > 1 implies xn = x which implies x(xn−1 − 1) = 0. Thus xn−1 = 1 since R=P is an integral domain. Now n > 1 implies x is a unit in R=P , with inverse xn−2. Thus R=P is a field so P is a maximal ideal. 2) Let A be a commutative ring with nilradical Nil(A). Show that the following are equivalent. i) A has exactly one prime ideal. ii) Every element of A is either a unit or a nilpotent. iii) A=Nil(A) is a field. Solution. i) ) ii). Suppose A has exactly one prime ideal P . By Proposition 8, on slide 10, lecture 12, the nilradical Nil(A) is the intersection of the prime ideals of A so Nil(A) = P . Thus every element of P is nilpotent. If x 2 A n P , then x cannot be contained in a maximal ideal of A since P is the unique maximal ideal of A. Thus x is a unit in A. ii) ) iii). Suppose x 2 A. Let x be the class of x in A=Nil(A). Now if x is nilpotent then x = 0 in A=Nil(A) and if x is not nilpotent then x is a unit in A which implies x is a unit in A=Nil(A). Thus A=Nil(A) is a field. iii) ) i). The assumption of iii) implies m = Nil(A) is a maximal ideal of A. By Proposition 8, slide 10 of Lecture 12, Nil(A) = \Q where the intersection is over all prime ideals of A. Thus m ⊂ Q for all prime ideals Q of A. Thus Q = m is the only prime ideal of A since m is maximal. Thus m is the unique prime ideal of A. 3) Let A be a (commutative) local ring, M and N be finitely generated A-modules. Prove that if M ⊗A N = 0 then M = 0 or N = 0. Hint: Apply the Corollary of Slide 6 of Lecture 10 when I = m is the maximal ideal of A. You may also use the following Lemma without proof, but you must explain when you use it. Lemma 0.1. Suppose that ' : A ! B is a homomorphism of commutative rings and E, E0 are two A-modules. Then there is an isomorphism of B-modules 0 ∼ 0 (E ⊗A B) ⊗B (E ⊗A B) = (E ⊗A E ) ⊗A B: A proof of this lemma can be found in Proposition 3, page AII.8.3 of Algebra, by N. Bourbaki. 1 Solution (to Problem 3). Let F = A=m which is a field. Suppose M ⊗A N = 0. Then ∼ 0 = (M ⊗A N) ⊗A A=m = (M ⊗A A=m) ⊗A=m (N ⊗A A=m) by the Lemma. But since F is a field, M ⊗A A=m and N ⊗A A=m are vector spaces over F and thus are free F -modules. M is a finitely generated F -module since the images of a finite set of generators of M as an A-module in M ⊗A A=m are generators of M ⊗A A=m as an A=m-module. Similarly, N ⊗A A=m is a finitely generated A=m-module. Thus both M ⊗A A=m and N ⊗A A=m have finite ranks. Thus 0 = rank((M ⊗A A=m) ⊗A=m (N ⊗A A=m)) = rank(M ⊗A A=m)rank(N ⊗A A=m) by the corollary on slide 20 of Lecture 9. Thus one of M ⊗A A=m or N ⊗A A=m has rank 0 and is thus zero. Without loss of generality, M ⊗A A=m = 0. Now ∼ M ⊗A A=m = M=mM by the corollary of slide 6 if Lecture 10. M=mM = 0 implies mM = M. Since A is a local ring with maximal ideal m (so that m is the Jacobson radical of A) we have that M = 0 by Nakayama's Lemma (Proposition 16, slide 5 of Lecture 13). 4) Let A be a commutative ring, M be a finitely generated A-module and ' : M ! An be a surjective homomorphism. Show that Ker ' is a finitely generated A-module. Solution. We have a short exact sequence of A-modules ' (1) 0 ! Ker ' !ι M ! An ! 0: An is a projective module over A since it is a free module (by the theorem on slide 17 of Lecture 5). Thus (1) is split exact by the theorem on slide 19 of Lecture 5. Thus there is a homomorphism : M ! Ker ' such that ι = idKer ' by the theorem on slide 3 of Lecture 5. Thus is surjective. Since M is a finitely generated A-module its image Ker ' is thus a finitely generated A-module. Explicitely, if y1; : : : ; yk generate M as an A-module, then (y1); : : : ; (yk) generate Ker ' as an A-module. 5) Let A be a commutative ring, M be an A-module and I an ideal of A. Suppose that Mm = 0 for all maximal ideals m which contain I. Prove that M = IM. ∼ Solution. Suppose m is a maximal ideal of A. Then (M=IM)m = Mm=(IM)m by Proposi- tion 20, slide 2 of Lecture 14. Suppose I 6⊂ m. Then there exists y 2 I \ (A n m). Suppose z z 1 yz x 2 Mm. Then x = s for some z 2 M and s 2 A n m and so x = s = y s 2 (IM)m. Thus (IM)m = Mm and so (M=IM)m = 0. Now suppose I ⊂ M. Then Mm = 0 by assumption and so (M=IM)m = 0. Thus (M=IM)m = 0 for all maximal ideals m of A and so M=IM = 0 by Proposition 22, slide 5 of Lecture 14. Thus M = IM. 2.

View Full Text

Details

  • File Type
    pdf
  • Upload Time
    -
  • Content Languages
    English
  • Upload User
    Anonymous/Not logged-in
  • File Pages
    2 Page
  • File Size
    -

Download

Channel Download Status
Express Download Enable

Copyright

We respect the copyrights and intellectual property rights of all users. All uploaded documents are either original works of the uploader or authorized works of the rightful owners.

  • Not to be reproduced or distributed without explicit permission.
  • Not used for commercial purposes outside of approved use cases.
  • Not used to infringe on the rights of the original creators.
  • If you believe any content infringes your copyright, please contact us immediately.

Support

For help with questions, suggestions, or problems, please contact us